Login

Welcome, Guest. Please login or register.

April 27, 2024, 09:54:48 pm

Author Topic: VCE Physics Question Thread!  (Read 609911 times)  Share 

0 Members and 4 Guests are viewing this topic.

BlinkieBill

  • Forum Regular
  • **
  • Posts: 86
  • Respect: +13
Re: VCE Physics Question Thread!
« Reply #1800 on: June 04, 2017, 12:53:14 pm »
0
Hey
I am doing a projectile distance practical for the epi. We're shooting nerf gun bullets at different angles and heights above the ground.
I was wondering what a good research question/aim would be for it. My teacher said something about terminal velocity and comparing practical results to theoretical (using formulas) results, but I'm not too sure about the research question still.

Any help is appreciated. Thanks
"One thing which sounds obvious - but is terribly overlooked - is that you have to be willing to work hard, no matter how bright you are." Dr Catherine Krupnick

2016: Mathematical Methods 42 | LOTE (CCAFL) - Punjabi 37
2017: English 41 | Chemistry 38 | Further Mathematics 50 (Premier's) | Specialist Mathematics 41 | Physics 45
ATAR 99.40

2018-2020: BMedRes @ UTAS

Offering online tutoring at only $35/hr PM me!

Selling Further Maths CORE MODULES NOTES for only $10 a module!! PM me for more information!

Guideme

  • Forum Regular
  • **
  • Posts: 87
  • Help me get the ATAR i want! :)
  • Respect: 0
Re: VCE Physics Question Thread!
« Reply #1801 on: June 05, 2017, 09:38:24 pm »
0
Please help me with these two questions Thank you a lot!
In general more heat is lost through the roof of a house then through the walls. Explain why this is so?

Often weather reports for skiing resorts will use the term 'wind chill factor'/ Explain the meaning of this term using appropriate thermodynamics principles and mechanisms
:0 :)

TooLazy

  • Trendsetter
  • **
  • Posts: 104
  • 2014 Forest Hills Drive
  • Respect: +23
Re: VCE Physics Question Thread!
« Reply #1802 on: June 09, 2017, 01:33:26 pm »
+1
I just want to confirm, when an object is decelerating the net force is in the direction opposite to its movement?
Future engineer

Alwin

  • Victorian
  • Forum Leader
  • ****
  • Posts: 838
  • Respect: +241
Re: VCE Physics Question Thread!
« Reply #1803 on: June 10, 2017, 10:37:32 am »
+2
Please help me with these two questions Thank you a lot!
In general more heat is lost through the roof of a house then through the walls. Explain why this is so?

Often weather reports for skiing resorts will use the term 'wind chill factor'/ Explain the meaning of this term using appropriate thermodynamics principles and mechanisms

If you think about how heat would travel (does it rise or does it fall) I think you'll be able to see why more heat is lost at the roof.
In terms of the wind chill factor, it comes down to the idea that you have air flowing over your skin rather than being stationary around you. A loose analogy would be letting your coffee cool down by letting it rest, or stirring your coffee (making it cool faster). Hopefully you should be able to take it from here :)

I just want to confirm, when an object is decelerating the net force is in the direction opposite to its movement?

 Yup! That would be correct :)
2012:  Methods [48] Physics [49]
2013:  English [40] (oops) Chemistry [46] Spesh [42] Indo SL [34] Uni Maths: Melb UMEP [4.5] Monash MUEP [just for a bit of fun]
2014:  BAeroEng/BComm

A pessimist says a glass is half empty, an optimist says a glass is half full.
An engineer says the glass has a safety factor of 2.0

Mattjbr2

  • Trendsetter
  • **
  • Posts: 165
  • Respect: +5
Re: VCE Physics Question Thread!
« Reply #1804 on: June 12, 2017, 06:45:19 pm »
0
http://i.imgur.com/XtCj6OP.png
Is the book's answer correct? Doesn't the flux increase when you increase the number of turns of wire? I'm getting conflicting information from multiple places: https://physics.stackexchange.com/questions/219881/is-the-number-of-turns-in-a-loop-part-of-the-magnetic-flux the dude says N (number of turns of wire) affects ϕ. Also: http://i.imgur.com/hzwGz0n.png the book even plainly states that. However, nowhere in the book is N included in the ϕ=BA equation.
What's going on!?
2017: Further (41)
2018: English Language - Specialist - Methods (40) - Chemistry
2019-2021: Biomedicine at Melbourne Uni

Insta: https://www.instagram.com/mattjbr2/

Mattjbr2

  • Trendsetter
  • **
  • Posts: 165
  • Respect: +5
Re: VCE Physics Question Thread!
« Reply #1805 on: June 13, 2017, 06:09:56 am »
0
Bump. Sac in 3 hours  ::)
2017: Further (41)
2018: English Language - Specialist - Methods (40) - Chemistry
2019-2021: Biomedicine at Melbourne Uni

Insta: https://www.instagram.com/mattjbr2/

sweetiepi

  • National Moderator
  • ATAR Notes Legend
  • *****
  • Posts: 4767
  • "A Bit of Chaos" (she/they)
  • Respect: +3589
Re: VCE Physics Question Thread!
« Reply #1806 on: June 13, 2017, 08:03:22 am »
0
http://i.imgur.com/XtCj6OP.png
Is the book's answer correct? Doesn't the flux increase when you increase the number of turns of wire? I'm getting conflicting information from multiple places: https://physics.stackexchange.com/questions/219881/is-the-number-of-turns-in-a-loop-part-of-the-magnetic-flux the dude says N (number of turns of wire) affects ϕ. Also: http://i.imgur.com/hzwGz0n.png the book even plainly states that. However, nowhere in the book is N included in the ϕ=BA equation.
What's going on!?

Hey, sorry for my late reply,
I'd definitely go with the with equation ϕ=nBA because the number of coils does increase the flux generated.
As there is the new study design, books are churned out very rapidly and generally contain a few mistakes. :)
2017-2019: Bachelor of Pharmaceutical Science (Formulation Science)
2020: Bachelor of Pharmaceutical Science (Honours) Read my uni journey here!

Syndicate

  • Forum Leader
  • ****
  • Posts: 797
  • Hard work beats Talent
  • Respect: +139
Re: VCE Physics Question Thread!
« Reply #1807 on: June 14, 2017, 09:23:56 pm »
+3
Hey, sorry for my late reply,
I'd definitely go with the with equation ϕ=nBA because the number of coils does increase the flux generated.
As there is the new study design, books are churned out very rapidly and generally contain a few mistakes. :)
The number of coils doesn't exactly effect the magnetic flux as the area remains the same (from a VCE point of a view, you might be correct from a first year perspective, but I am not really sure of that) . Magnetic flux is defined as the amount of magnetic fields going through a certain area, so the amount of coils doesn't affect it.
(This is directly out of my book as well, so I am not really sure if I am correct either  :P)
« Last Edit: June 14, 2017, 09:25:49 pm by Syndicate »
2017: Chemistry | Physics | English | Specialist Mathematics | Mathematics Methods
2018-2020 : Bachelor of Biomedicine at University of Melbourne

Physics Guide 2017

zsteve

  • ATAR Notes VIC MVP - 2016
  • Forum Leader
  • ****
  • Posts: 748
  • The LORD is my shepherd, I shall not want - Ps. 23
  • Respect: +218
Re: VCE Physics Question Thread!
« Reply #1808 on: June 15, 2017, 08:19:29 pm »
+2
From the way it's put in VCE, to the best of my understanding:
- Flux is related to the total 'amount' of field passing through a given surface area (so )
- The presence of multiple coils does affect the current/potential produced, but this is accounted for when dealing with Faraday's Law
~~ rarely checking these forums these days ~~

2015: Specialist [47] | Methods [48] | Chemistry [50] | Physics [48] | English Language [46] | UMEP Mathematics [5.0] | ATAR - 99.95
Premier's Award Recipient 2016: Top All-Round VCE High Achiever
2016-2019: University of Melbourne : Bachelor of Science (Biochemistry & Molecular Biology), Diploma in Mathematics (Applied)
2019-: University of British Columbia

Mattjbr2

  • Trendsetter
  • **
  • Posts: 165
  • Respect: +5
Re: VCE Physics Question Thread!
« Reply #1809 on: June 17, 2017, 05:43:21 pm »
0
Therefore is my book's answer correct or incorrect?
2017: Further (41)
2018: English Language - Specialist - Methods (40) - Chemistry
2019-2021: Biomedicine at Melbourne Uni

Insta: https://www.instagram.com/mattjbr2/

Syndicate

  • Forum Leader
  • ****
  • Posts: 797
  • Hard work beats Talent
  • Respect: +139
Re: VCE Physics Question Thread!
« Reply #1810 on: June 17, 2017, 05:53:25 pm »
0
Therefore is my book's answer correct or incorrect?

Your book is right with the equation part, but, for question 2 (faraday's experiment question), I believe he used a coil with many turns as it increases the strength of static magnetic field around the coil, which thereby results in a greater EMF induced. They didn't have modern technology to measure the induced EMF, so if only a small EMF was induced, then they wouldn't really be able to calculate it (or know if something really happened or not).
2017: Chemistry | Physics | English | Specialist Mathematics | Mathematics Methods
2018-2020 : Bachelor of Biomedicine at University of Melbourne

Physics Guide 2017

Mattjbr2

  • Trendsetter
  • **
  • Posts: 165
  • Respect: +5
Re: VCE Physics Question Thread!
« Reply #1811 on: June 17, 2017, 06:22:55 pm »
0
Your book is right with the equation part, but, for question 2 (faraday's experiment question), I believe he used a coil with many turns as it increases the strength of static magnetic field around the coil, which thereby results in a greater EMF induced. They didn't have modern technology to measure the induced EMF, so if only a small EMF was induced, then they wouldn't really be able to calculate it (or know if something really happened or not).
What about question 3? The back of the book gives 3.0x10^-5 Wb but the worked solutions give 3.0x10^-3 Wb and neither took into consideration the 50 turns.
2017: Further (41)
2018: English Language - Specialist - Methods (40) - Chemistry
2019-2021: Biomedicine at Melbourne Uni

Insta: https://www.instagram.com/mattjbr2/

Syndicate

  • Forum Leader
  • ****
  • Posts: 797
  • Hard work beats Talent
  • Respect: +139
Re: VCE Physics Question Thread!
« Reply #1812 on: June 17, 2017, 09:03:34 pm »
0
What about question 3? The back of the book gives 3.0x10^-5 Wb but the worked solutions give 3.0x10^-3 Wb and neither took into consideration the 50 turns.

It is most likely a misprint. The solutions with 3.0 x 10^-3 Wb is the correct one.
2017: Chemistry | Physics | English | Specialist Mathematics | Mathematics Methods
2018-2020 : Bachelor of Biomedicine at University of Melbourne

Physics Guide 2017

gamma032

  • Adventurer
  • *
  • Posts: 19
  • Respect: +2
Re: VCE Physics Question Thread!
« Reply #1813 on: June 18, 2017, 06:28:42 pm »
0
Hi everyone!

I've been trying to calculate the uncertainty in a couple of calculations, but I'll just present the hardest.

I'm finding the gravitational potential energy (m*g*h) of an object, and I measured:
  • The measurement uncertainty in the mass to be ±0.005kg
  • The uncertainty in the height to be ±0.0005m

I'm using g=9.8.

What would the final uncertainty of the GPE calculations be?
2016: Business Management [40]
2017: Spesh [37] Methods [40], Physics [42], Accounting [47], English [36]
ATAR: 97.70
2018-2020: BSci, Computing & Software Systems @ Unimelb

zsteve

  • ATAR Notes VIC MVP - 2016
  • Forum Leader
  • ****
  • Posts: 748
  • The LORD is my shepherd, I shall not want - Ps. 23
  • Respect: +218
Re: VCE Physics Question Thread!
« Reply #1814 on: June 18, 2017, 07:51:54 pm »
+1
Hi everyone!

I've been trying to calculate the uncertainty in a couple of calculations, but I'll just present the hardest.

I'm finding the gravitational potential energy (m*g*h) of an object, and I measured:
  • The measurement uncertainty in the mass to be ±0.005kg
  • The uncertainty in the height to be ±0.0005m

I'm using g=9.8.

What would the final uncertainty of the GPE calculations be?


From memory, what one must do when adding two quantities is to deal with fractional error. Consider c=a+b. Then the fractional error in c is given by:



Where a, b, c are the mean values of a, b, c; and the Delta values are the error margins.

This is also how we deal with errors for division :)
~~ rarely checking these forums these days ~~

2015: Specialist [47] | Methods [48] | Chemistry [50] | Physics [48] | English Language [46] | UMEP Mathematics [5.0] | ATAR - 99.95
Premier's Award Recipient 2016: Top All-Round VCE High Achiever
2016-2019: University of Melbourne : Bachelor of Science (Biochemistry & Molecular Biology), Diploma in Mathematics (Applied)
2019-: University of British Columbia